2014 dxdy logo

Научный форум dxdy

Математика, Физика, Computer Science, Machine Learning, LaTeX, Механика и Техника, Химия,
Биология и Медицина, Экономика и Финансовая Математика, Гуманитарные науки


Правила форума


В этом разделе нельзя создавать новые темы.

Если Вы хотите задать новый вопрос, то не дописывайте его в существующую тему, а создайте новую в корневом разделе "Помогите решить/разобраться (М)".

Если Вы зададите новый вопрос в существующей теме, то в случае нарушения оформления или других правил форума Ваше сообщение и все ответы на него могут быть удалены без предупреждения.

Не ищите на этом форуме халяву, правила запрещают участникам публиковать готовые решения стандартных учебных задач. Автор вопроса обязан привести свои попытки решения и указать конкретные затруднения.

Обязательно просмотрите тему Правила данного раздела, иначе Ваша тема может быть удалена или перемещена в Карантин, а Вы так и не узнаете, почему.



Начать новую тему Ответить на тему На страницу Пред.  1, 2
 
 
Сообщение04.12.2006, 20:49 
Заслуженный участник


28/10/05
1368
Rassmotrite $ \int\limits_{-\infty}^{\infty} e^{-at^2} dt=I(a)=\sqrt{\frac{{\pi}}{a}}$, prodifferentsiruite $\frac{dI(a)}{da}=\int\limits_{-\infty}^{\infty} - t^2 e^{-at^2} dt=-\frac{1}{2}a^{-\frac{3}{2}}\sqrt{\pi}$. Vot i reshenie v odnu stroku.

 Профиль  
                  
 
 
Сообщение04.12.2006, 21:15 


03/12/06
9
Ну, да. Только вот $\sqrt{2\pi} * 2/ \sqrt{2\pi} = 2$, а должно быть 4...

 Профиль  
                  
 
 
Сообщение04.12.2006, 21:23 
Заслуженный участник


28/10/05
1368
hicks писал(а):
Ну, да. Только вот $\sqrt{2\pi} * 2/ \sqrt{2\pi} = 2$, а должно быть 4...


Ya ne smotrela vichisleniya vseh uchastnikov. U Vas bil konkretniy vopros.

hicks писал(а):
Вот, что у меня получается, подскажите, пожалуйста, как вычислить этот интеграл.

$\frac {1} {\sqrt{2\pi}} \int\limits_{-\infty}^{\infty} 2t^2 e^\frac {-t^2} {2} dt$


Vozmojniy sposob resheniya Vam podskazali dvajdi.

 Профиль  
                  
 
 
Сообщение04.12.2006, 22:08 
Заслуженный участник
Аватара пользователя


09/10/05
1142
"Mathematica" даёт тоже $$\int\limits_{-\infty}^{\infty} t^2 \cdot e^{-\frac {t^2} 2} dt = \sqrt {2 \pi}$$ :flood2:

А можно посмотреть, как Вы пришли к этому интегралу? :roll:

Добавлено спустя 4 минуты 14 секунд:

hicks писал(а):
К тому же, у Вас, в итоге, получается $\sqrt{2\pi}$, а должно быть 4, что легко проверить:

$e^\frac {-(x-a)^2} {2\sigma^2} = e^\frac {-(x+3)^2} {2*2^2} $

откуда $\sigma=2$, а дисперсия равна $\sigma^2$, и, след-но, получается 4.


А понятно, но в итоге у меня получается 2, а не $$\sqrt {2\pi}$$

Дело в том, что константы были вынесены до интегрирования, поэтому корни сократятся..

 Профиль  
                  
 
 
Сообщение04.12.2006, 22:41 


03/12/06
9
Ах, я в том интеграле забыл скобки поставить... должно быть так:
$\frac {1} {\sqrt{2\pi}} \int\limits_{-\infty}^{\infty} (2t)^2 e^{\frac {-t^2} {2}} dt$
Тогда получится правильно - 4

А интеграл я получил так:
$D=\int\limits_{-\infty}^{\infty} (x-M)^2 f(x) dx$
$D=\int\limits_{-\infty}^{\infty} (x+3)^2 \frac {1} {2\sqrt{2\pi}} e^{\frac {-(x+3)^2} {2*2^2}} dx$
И после замены переменной $t=\frac {x+3} {2}, dx=2dt$ получился тот интеграл.

И последнее, что меня интересует - это вероятность попадания СВ в интервал (0;3). По формуле получается это:
$\int\limits_{-\infty}^{\infty} f(x) dx$
$\int\limits_{0}^{3} \frac {1} {\sqrt{8\pi}} e^{\frac {-(x+3)^2} {2*2^2}}}dx$
С ним-то, что делать?
PS. Не удивляйтесь, интегралы - это мой самый страшный кошмар...

 Профиль  
                  
 
 
Сообщение04.12.2006, 22:52 
Заслуженный участник
Аватара пользователя


09/10/05
1142
Я делаю Вам полный расклад:

$$Var(X) = E(X- \mu)^2 = \frac 1 {\sigma \sqrt{2\pi}} \int\limits_{-\infty}^{\infty}$$ $$ \left(x - \mu \right)^2 e^{-\frac{(x-\mu)^2} {2\sigma^2}} dx$$

На этом месте я делаю сл подстановку: $$ y = \frac{x - \mu} {\sigma} $$
Теперь возвожу в квадрат: $$ y^2 = \frac {\left( x - \mu\right)^2} {\sigma^2} $$
Переношу сигму к игрику и делаю подстановку в интеграле:


$$Var(X) = E(X- \mu)^2 = \frac 1 {\sigma \sqrt{2\pi}} \int\limits_{-\infty}^{\infty}$$ $$ \left( y \cdot \sigma \right)^2 e^{-\frac{y^2} {2} dx$$

Выношу все константы и сокращаю сигму из знаменателя с квадратом в числителе. Окончательно:

$$\frac {\sigma}{\sqrt{2\pi}} \int\limits_{-\infty}^{\infty}y^2 e^{-\frac{y^2} {2} }dx$$

Вообще остаётся не совсем понтно, как Вы нашли $$2t^2$$... Может-быть $$\left( 2t \right) ^2 = 4 t^2$$?

Добавлено спустя 53 секунды:

ok

Добавлено спустя 7 минут 15 секунд:

Прошу прощения, у меня не совсем правильно там, поскольку я копиравала уже все интегралы и забыла сделать подстановку для $$dx$$ :x

 Профиль  
                  
 
 
Сообщение04.12.2006, 22:53 


03/12/06
9
Большое спасибо Вам за труды, но с дисперсией я уже разобрался. Только попадание в интервал осталось...

 Профиль  
                  
 
 
Сообщение04.12.2006, 22:59 
Заслуженный участник
Аватара пользователя


09/10/05
1142
С Вашим последним интегралом та-же история, НО Вам нужно будет изменить границы интегрирования. Там надо будет сделать сл подстановку: $$ y = \frac {x + 3} 4$$. Вам надо разрешить это относительно х (область определения) и подставить в границы интегрирования...

 Профиль  
                  
 
 
Сообщение05.12.2006, 09:59 


03/12/06
9
Вот чему, должна быть равна вероятность попадания в интервал:
$P(x_1<\xi<x_2)=\phi(x_2)-\phi(x_1)$
$\frac {1} {\sqrt{2\pi}} \int\limits_{-\infty}^{\frac{x_2-m} {\sigma}} e^{\frac{-t^2} {2}} dt - \frac {1} {\sqrt{2\pi}} \int\limits_{-\infty}^{\frac{x_1-m} {\sigma}} e^{\frac{-t^2} {2}} dt$
$\frac {1} {\sqrt{2\pi}} \int\limits_{-\infty}^{3} e^{\frac{-t^2} {2}} dt - \frac {1} {\sqrt{2\pi}} \int\limits_{-\infty}^{3/2} e^{\frac{-t^2} {2}} dt$
Но эти интегралы не берет даже Mathematica, что уж говорить обо мне... Я уже совсем запутался. Объясните, пожалуйста, как же это решается.

 Профиль  
                  
 
 
Сообщение05.12.2006, 10:07 
Супермодератор
Аватара пользователя


29/07/05
8248
Москва
Эти интегралы не берутся, это известно. В любой книге по теории вероятностей в конце приводится таблица значений указанной функции, найденных численно. Интеграл до 3 равен $0.998650$, а интеграл до 1.5 равен $0.933193$. Разумеется, это приближенные значения.

 Профиль  
                  
 
 
Сообщение05.12.2006, 10:24 


03/12/06
9
Большое, Вам, спасибо!

 Профиль  
                  
 
 
Сообщение05.12.2006, 13:47 
Заслуженный участник
Аватара пользователя


09/10/05
1142
hicks писал(а):
$\frac {1} {\sqrt{2\pi}} \int\limits_{-\infty}^{3} e^{\frac{-t^2} {2}} dt - \frac {1} {\sqrt{2\pi}} \int\limits_{-\infty}^{3/2} e^{\frac{-t^2} {2}} dt$
Но эти интегралы не берет даже Mathematica....


Берёт, берёт :lol: Но там есть фокус! Вы когда этот интеграл в Mathematica делаете, поставьте сразу после него вот такое обозначение:
Код:
... //N
:wink:

 Профиль  
                  
Показать сообщения за:  Поле сортировки  
Начать новую тему Ответить на тему  [ Сообщений: 27 ]  На страницу Пред.  1, 2

Модераторы: Модераторы Математики, Супермодераторы



Кто сейчас на конференции

Сейчас этот форум просматривают: нет зарегистрированных пользователей


Вы не можете начинать темы
Вы не можете отвечать на сообщения
Вы не можете редактировать свои сообщения
Вы не можете удалять свои сообщения
Вы не можете добавлять вложения

Найти:
Powered by phpBB © 2000, 2002, 2005, 2007 phpBB Group